Difference between revisions of "2022 AMC 12A Problems/Problem 18"

(Created page with "==Problem== Let <math>T_k</math> be the transformation of the coordinate plane that first rotates the plane <math>k</math> degrees counter-clockwise around the origin and the...")
 
(Redirected page to 2022 AMC 10A Problems/Problem 18)
(Tag: New redirect)
 
(7 intermediate revisions by 5 users not shown)
Line 1: Line 1:
==Problem==
+
#redirect [[2022 AMC 10A Problems/Problem 18]]
 
 
Let <math>T_k</math> be the transformation of the coordinate plane that first rotates the plane <math>k</math> degrees counter-clockwise around the origin and then reflects the plane across the <math>y</math>-axis. What is the least positive
 
integer <math>n</math> such that performing the sequence of transformations <math>T_1, T_2, T_3, \cdots, T_n</math> returns the point <math>(1,0)</math> back to itself?
 
 
 
==Solution==
 
 
 
https://youtu.be/QQrsKTErJn8
 
 
 
(Professor Chen Education Palace, www.professorchenedu.com)
 

Latest revision as of 05:17, 19 November 2022